LSAT and Law School Admissions Forum

Get expert LSAT preparation and law school admissions advice from PowerScore Test Preparation.

 Administrator
PowerScore Staff
  • PowerScore Staff
  • Posts: 8916
  • Joined: Feb 02, 2011
|
#26974
Complete Question Explanation

StrengthenX. The correct answer choice is (E)

In this stimulus the author presents a very basic assertion: even when clearly overstaffed, midlevel managers are unlikely to suggest reductions. Since this stimulus is followed by a Strengthen Except question, the four incorrect answer choices will strengthen the author’s assertion, and the correct answer choice will not strengthen the author’s argument.

Incorrect answer choices (A) and (B) both strengthen the author’s assertion by pointing out incentives that mid-level managers have to avoid the suggestion of staffing cuts. Answer choice (A) motivates with the incentive of increased pay with more staff, and answer choice (B) does the same, but with the incentive of decreased workload.

Answer choices (C) and (D) both strengthen the author’s argument by listing some of the benefits of maintaining staff even when overstaffed; (C) discussed maintaining morale, and (D) points out the lack of predictability that may necessitate overstaffing.

Answer choice (E): This is the correct answer choice. Correct answer choice (E) is completely irrelevant to the topic of midlevel managers and their staffing decisions. The fact that large corporations provide individuals with incentives to retire has no effect on the author’s argument, so this is the correct answer choice to this Strengthen Except question.
 Blueballoon5%
  • Posts: 156
  • Joined: Jul 13, 2015
|
#41077
Hello! Why is answer choice E irrelevant to the discussion? I thought that it might weaken the argument, since managers may want to actually reduce staff in the hopes of retiring soon. Thanks!!
 Claire Horan
PowerScore Staff
  • PowerScore Staff
  • Posts: 408
  • Joined: Apr 18, 2016
|
#42184
Hi Blueballoon,

Sure, I think that could be a fair reading if you read the word "staff" to include the manager him/herself. Those who argue that (E) is irrelevant are likely assuming, I think reasonably, that "staff" refers to the workers the manager supervises rather than the manager him/herself. In any case, if (E) weakens the argument, then it doesn't strengthen it, so (E) is still the correct answer.

Get the most out of your LSAT Prep Plus subscription.

Analyze and track your performance with our Testing and Analytics Package.